LSAT and Law School Admissions Forum

Get expert LSAT preparation and law school admissions advice from PowerScore Test Preparation.

 Administrator
PowerScore Staff
  • PowerScore Staff
  • Posts: 8917
  • Joined: Feb 02, 2011
|
#47138
Complete Question Explanation
(The complete setup for this game can be found here: lsat/viewtopic.php?t=2909)

The correct answer choice is (B)

The question stem kindly provides the complete color combination in one of the windows—P, R, and O—and then asks you to identify a complete color combination that could be used in another window. The initial setup appears as follows, with no window numbers specified:
D10_game #2_#10_diagram 1.png
Of course, the first rule specifies that one of the windows must have G and P, and so that fact can be added to the diagram:
D10_game #2_#10_diagram 2.png
Y must also be used in one of the windows, and from the third rule Y cannot be used with G or O, and so Y must be used in the remaining window, along with P:
D10_game #2_#10_diagram 3.png
With the diagram above—which shows the minimum color usage in the other two windows—answer choices (C), (D), and (E) can each be eliminated. This is because each contains exactly two colors, and any two color window in under the above scenario would have to be GP or YP. As neither of those color combinations is listed in any of those three answers, each can be eliminated.

Answer choice (A) can also be eliminated because it does not contain P, and the diagram shows that each window must contain P. Thus, answer choice (B) is the correct answer, and under answer choice (B), the “third” window would have the remaining R:
D10_game #2_#10_diagram 4.png
You do not have the required permissions to view the files attached to this post.
 mjacobson
  • Posts: 2
  • Joined: Jun 25, 2020
|
#76524
I am having a hard time understanding why purple and orange can go together given rule four...would you be able to explain that to me? This was a tough game overall for me!
 Adam Tyson
PowerScore Staff
  • PowerScore Staff
  • Posts: 5153
  • Joined: Apr 14, 2011
|
#76560
You've just asked about one of the most troubling and dangerous rules in all of logic games, mjacobson! I'll refer you to this blog post for more about it:

https://blog.powerscore.com/lsat/the-mo ... rule-lsat/

For now, here's the short version. The fourth rule is that if you do NOT have purple, you must have orange. The sufficient condition is NOT having purple. So what do we know about orange when a window DOES have purple, and the sufficient condition of not having purple fails to occur? We know nothing about orange in that case, because the rule requiring orange isn't triggered. Maybe we have orange, and maybe not! It's only when purple is missing that we are guaranteed something about orange, that it must be present.

The purple and orange rule means that every window must have purple, or orange, or both. The only thing it prevents is a window that has neither. And that's why we can infer that a window that has yellow MUST also have purple, because yellow knocks out orange, and we have to have either purple or orange in every window. No orange? Bring in the purple!

Check out that blog post and see if it rocks your world and brings your gaming to the next level. Have fun!
 mjacobson
  • Posts: 2
  • Joined: Jun 25, 2020
|
#76583
Thank you so much for the explanation and blog post...they really helped a lot! :-D

Get the most out of your LSAT Prep Plus subscription.

Analyze and track your performance with our Testing and Analytics Package.